Please help and show work pls
noah and emma are standing on opposite sides of a 43.3 ft tree looking up at noah’s cat, which is perched at the very top. they are separated by a horizontal distance of 100 ft. the distance from emma to the cat is 50 ft and her angle of elevation to see the cat is 60°. the distance from noah to the cat is 86.6 ft and his angle of elevation to see the cat is 30°.

(a) identify three different trigonometric ratios that could have been used to find the distance x between emma and the base of the tree. for each trigonometric ratio, determine the distance. round to the nearest whole number.

(b) use the pythagorean theorem to find the distance x between emma and the base of the tree. round to the nearest whole number.

Answers

Answer 1

(a) The three different trigonometric ratios that could have been used to find the distance x between Emma and the base of the tree are sin(60°) = 43.3/50, cos(60°) = x / 50, and tan(60°) = 43.3 / x. The distance is 25 ft.

(b) Using the Pythagorean theorem, the distance x between Emma and the base of the tree is 25 ft.

(a) We can use sine, cosine, and tangent trigonometric ratios to find the distance x between Emma and the base of the tree.

1. Sine:
sin(60°) = opposite/hypotenuse = (tree height) / 50
tree height = 50 * sin(60°) = 43.3 ft (since it's given that the tree is 43.3 ft tall)

2. Cosine:
cos(60°) = adjacent/hypotenuse = x / 50
x = 50 * cos(60°) = 25 ft

3. Tangent:
tan(60°) = opposite/adjacent = (tree height) / x
x = (tree height) / tan(60°) = 43.3 / tan(60°) ≈ 25 ft

(b) To find the distance x between Emma and the base of the tree using the Pythagorean theorem, we can consider the triangle formed by Emma, the base of the tree, and the top of the tree.

Let's call the distance from the base of the tree to the top of the tree (tree height) y.

Emma's distance to the cat (50 ft) is the hypotenuse, the distance x is one leg, and the tree height y (43.3 ft) is the other leg of the right triangle.

Using the Pythagorean theorem: x² + y² = hypotenuse²
x² + 43.3² = 50²
x² + 1874.89 = 2500
x² = 625.11
x ≈ 25 ft

So, the distance between Emma and the base of the tree is approximately 25 ft.

Learn more about trigonometric ratios here: https://brainly.com/question/10417664

#SPJ11


Related Questions

An online furniture store sells chairs for $50 each and tables for $250 each. Every day, the store can ship no more than 26 pieces of furniture and must sell a minimum of $1900 worth of chairs and tables. Also, the store must sell a minimum of 14 tables. If a represents the number of tables sold and y represents the number of chairs sold, write and solve a system of inequalities graphically and determine one possible solution.​

Answers

Answer:

9, 10, 11, 12, 13.

Step-by-step explanation:

All possible values for the number of tables that the store must sell in order to meet the requirements are 9, 10, 11, 12, 13

Write the repeating decimal as a geometric series. 0,216

Answers

the repeating decimal 0.216 can be written as the geometric series: 0.216 = 216/990.

To write the repeating decimal 0.216 as a geometric series, we first need to express it in the form of a sum of a geometric series.

The decimal 0.216 repeats every three digits, so we can break it down as follows:

0.216 = 0.2 + 0.01 + 0.006 + 0.0002 + 0.00001 + 0.000006 + ...

Now, we can write this as a sum of a geometric series with the first term (a) and the common ratio (r):

a = 0.2
r = 0.01 (because each term is 1/100 of the previous term)

Thus, the geometric series for the repeating decimal 0.216 is:

0.216 = 0.2 + 0.2(0.01) + 0.2(0.01)^2 + 0.2(0.01)^3 + ...

The formula for the sum of an infinite geometric series is S = a / (1 - r), where S is the sum, a is the first term, and r is the common ratio.

Using the values for a and r, we can find the sum of the series:

S = 0.2 / (1 - 0.01) = 0.2 / 0.99 = 216/990.

To learn more about geometric series click here

brainly.com/question/21087466

#SPJ11

Please help


Michael thought he could only run 5 laps around the track but he was actually able to run 8 laps what was his percent error round to the nearest percent

Answers

To calculate the percent error, we need to use the following formula:

percent error = (|measured value - actual value| / actual value) x 100%

1. Determine the difference between the actual value (8 laps) and the estimated value (5 laps).
Actual value = 8 laps
Estimated value = 5 laps
Difference = Actual value - Estimated value = 8 - 5 = 3 laps

2. Divide the difference by the actual value:
Percent error (decimal) = Difference / Actual value = 3 laps / 8 laps = 0.375

3. Convert the decimal to a percentage by multiplying by 100:
Percent error = 0.375 * 100 = 37.5%

4. Round to the nearest percent:
Percent error ≈ 38%

So, Michael's percent error in estimating his laps around the track was approximately 38%.

Learn more about percent error at https://brainly.in/question/2230099

#SPJ11

Sabine rode on a passenger train for 480 miles between 10:30 A. M. And 6:30 P. M. A friend in a different city

Answers

The speed of the train is 60 miles per hour.

Sabine travel 480 miles on a passenger train between 10:30 A.M. and 6:30 P.M. What is speed of train?

We calculate in two steps:

Calculate the speed of the train

To calculate the speed of the train, we need to use the formula:

Speed = Distance / Time

Here, the distance travelled by the train is 480 miles, and the time taken is 8 hours (from 10:30 A.M. to 6:30 P.M.). So, we can calculate the speed of the train as:

Speed = 480 miles / 8 hours

Speed = 60 miles per hour

Therefore, the speed of the train is 60 miles per hour.

Explain the solution

Sabine rode on a passenger train for 480 miles between 10:30 A.M. and 6:30 P.M.

To calculate the speed of the train, we used the formula Speed = Distance / Time, where Distance is 480 miles and Time is 8 hours (since the journey was between 10:30 A.M. and 6:30 P.M.).

Substituting the values, we get the speed of the train as 60 miles per hour.

This means that the train travelled at a speed of 60 miles per hour throughout the journey, covering a distance of 480 miles in 8 hours.

Learn more about speed

brainly.com/question/30462853

#SPJ11

Use logarithmic differentiation to find the derivative of the function y= x²/x y'(x)= 2 + 1 In x) x²

Answers

To use logarithmic differentiation to find the derivative of the function y = x²/x, we first take the natural logarithm of both sides:

ln(y) = ln(x²/x)

Using the properties of logarithms, we can simplify this to:

ln(y) = 2 ln(x) - ln(x)

Now we differentiate both sides with respect to x using the chain rule:

1/y * y' = 2/x - 1/x

Simplifying this expression, we get:

y' = y * (2/x - 1/x²)

Substituting back in the original expression for y, we have:

y' = x²/x * (2/x - 1/x²)

Simplifying further, we get: y' = 2x - 1/x

Therefore, the derivative of the function y = x²/x using logarithmic differentiation is y' = 2x - 1/x.

Learn more about logarithmic differentiation,

https://brainly.com/question/31299592

#SPJ11

The line on a coordinate plane makes an angle of depression 32 degrees. What is the slope of the line

Answers

The slope of the line on a coordinate plane that makes an angle of depression of 32 degrees is approximately 0.625.

To find the slope of the line on a coordinate plane that makes an angle of depression of 32 degrees,:

Step 1: Determine the angle of elevation. Since the angle of depression is 32 degrees, the angle of elevation is also 32 degrees, because they are alternate angles.

Step 2: Use the tangent function to find the slope. The tangent of an angle in a right triangle is equal to the ratio of the side opposite the angle (rise) to the side adjacent to the angle (run). In this case, the tangent of the angle of elevation (32 degrees) is equal to the slope of the line.

Step 3: Calculate the tangent of 32 degrees. Using a calculator or a trigonometric table, you can find that tan(32°) ≈ 0.625.

So, the slope of the line on a coordinate plane that makes an angle of depression of 32 degrees is approximately 0.625.

To know more about slope refer here:

https://brainly.com/question/16180119?#

#SPJ11

Is it Linear, exponential, Quadratic or neither

Answers

quadratic since numbers flip

TRUE or FALSE:


1. Each exterior angle of a regular hexagon is acute


2. The sum of the interior angles of a polygon is not necessarily a multiple of 180


3. In any polygon, the larger the number of vertices, the smaller the measure of an exterior angle

Answers

1. The statement "Each exterior angle of a regular hexagon is acute" is True.

2. The statement "The sum of the interior angles of a polygon is always a multiple of 180" is False.

3. The statement "In any polygon, the larger the number of vertices, the smaller the measure of an exterior angle" is True.

1. TRUE: Each exterior angle of a regular hexagon is acute.
A regular hexagon has six equal sides and six equal interior angles. The sum of the interior angles of a hexagon is (6-2) * 180 = 720 degrees. Since it's a regular hexagon, each interior angle is 720/6 = 120 degrees. The exterior angles are supplementary to the interior angles, so each exterior angle is 180 - 120 = 60 degrees. Since 60 degrees is less than 90 degrees, each exterior angle is acute.

2. FALSE: The sum of the interior angles of a polygon is always a multiple of 180.
The formula for the sum of the interior angles of a polygon is (n-2) * 180, where n is the number of vertices (or sides). As you can see, the result is always a multiple of 180.

3. TRUE: In any polygon, the larger the number of vertices, the smaller the measure of an exterior angle.
For a regular polygon, the measure of an exterior angle can be calculated as 360/n, where n is the number of vertices (or sides). As the number of vertices increases, the measure of an exterior angle decreases, since they are inversely proportional.

Learn more about "angle": https://brainly.com/question/25716982

#SPJ11

Regular quadrilateral prism has a height h = 11 cm and base edges b= 8cm. Find the sum of al edges

Answers

The sum of all edges of the regular quadrilateral prism is 108 cm.

To find the sum of all edges of a regular quadrilateral prism with height h = 11 cm and base edges b = 8 cm, follow these steps:

1. Determine the number of base edges: A quadrilateral has 4 edges, so there are 4 base edges for the top and 4 for the bottom, totaling 8 base edges.
2. Determine the number of height edges: There are 4 vertical edges connecting the top and bottom bases.
3. Add the number of base and height edges: 8 base edges + 4 height edges = 12 edges in total.
4. Calculate the sum of all edge lengths: (8 base edges (8 cm)) + (4 height edges (11 cm)) = 64 cm + 44 cm = 108 cm.

So, the sum of all edges of the regular quadrilateral prism is 108 cm.

To know more about "Quadrilateral prism" refer here:

https://brainly.com/question/9982469#

#SPJ11

A workplace gave an "employee culture survey" in which 500 employees rated their agreement with the statement, "i feel respected by those i work for. " rating frequency strongly agree 156 agree 114 neutral 99 disagree 88 strongly disagree 43 the relative frequency of people who strongly agree with the statement is __________

Answers

The relative frequency of people who strongly agree with the statement "I feel respected by those I work for" is 0.312, or 31.2%.

This means that out of the 500 employees surveyed, 156 strongly agreed with the statement. To find the relative frequency, you simply divide the number of people who strongly agree by the total number of people surveyed (156/500).

This result suggests that the majority of employees feel respected by their employers, which is a positive sign for the workplace culture.

However, it's important to note that there are still a significant number of employees who either disagree or feel neutral about this statement, indicating that there may be room for improvement in terms of fostering a more respectful and supportive work environment.

To know more about relative frequency  click on below link:

https://brainly.com/question/29739263#

#SPJ11

Solve the problem.
Find the area bounded by y = 3 / (√36-9x^2) • X = 0, y = 0, and x = 3. Give your answer in exact form.

Answers

To solve the problem, we first need to graph the equation y = 3 / (√36-9x^2) and find the points where it intersects the x-axis and y-axis.

To find the x-intercept, we set y = 0 and solve for x:
0 = 3 / (√36-9x^2)
0 = 3
This has no solution, which means that the graph does not intersect the x-axis.

To find the y-intercept, we set x = 0 and solve for y:
y = 3 / (√36-9(0)^2)
y = 3 / 6
y = 1/2
So the graph intersects the y-axis at (0, 1/2).

Next, we need to find the point where the graph intersects the vertical line x = 3. To do this, we substitute x = 3 into the equation y = 3 / (√36-9x^2):
y = 3 / (√36-9(3)^2)
y = 3 / (√-243)
This is undefined, which means that the graph does not intersect the line x = 3.

Now we can draw a rough sketch of the graph and the region bounded by the x-axis, the line x = 0, and the curve y = 3 / (√36-9x^2):

           |
    _______|
   /       |
  /        |
 /         |
/_________|
|         |

The area we want to find is the shaded region, which is bounded by the x-axis, the line x = 0, and the curve y = 3 / (√36-9x^2). To find the area, we need to integrate the equation y = 3 / (√36-9x^2) with respect to x from x = 0 to x = 3:

A = ∫(0 to 3) 3 / (√36-9x^2) dx

We can simplify this integral by using the substitution u = 3x, du/dx = 3, dx = du/3:

A = ∫(0 to 9) 1 / (u^2 - 36) du/3

Next, we use partial fractions to break up the integrand into simpler terms:

1 / (u^2 - 36) = 1 / (6(u - 3)) - 1 / (6(u + 3))

So we have:

A = ∫(0 to 9) (1 / (6(u - 3))) - (1 / (6(u + 3))) du/3

A = (1/6) [ln|u - 3| - ln|u + 3|] from 0 to 9

A = (1/6) [ln(6) - ln(12) - ln(6) + ln(6)]

A = (1/6) [ln(1/2)]

A = (-1/6) ln(2)

Therefore, the exact area bounded by y = 3 / (√36-9x^2), x = 0, y = 0, and x = 3 is (-1/6) ln(2).
To find the area bounded by y = 3 / (√36-9x^2), x = 0, y = 0, and x = 3, we can set up an integral to compute the definite integral of the function over the given interval [0, 3]. The integral will represent the area under the curve:

Area = ∫[0, 3] (3 / (√(36-9x^2))) dx

To solve the integral, perform a substitution:

Let u = 36 - 9x^2
Then, du = -18x dx

Now, we can rewrite the integral:

Area = ∫[-√36, 0] (-1/6) (3/u) du

Solve the integral:

Area = -1/2 [ln|u|] evaluated from -√36 to 0

Area = -1/2 [ln|0| - ln|-√36|]

Area = -1/2 [ln|-√36|]

Since the natural logarithm of a negative number is undefined, there's an error in the original problem. Check the problem's constraints and the given function to ensure accuracy before proceeding.

Learn more about graphs here: brainly.com/question/17267403

#SPJ11

Estimating Estimate to as many decimal places as your calculator will display by using Newton's method to solve the equation tan(x) = 0 with xo 3.

Answers

The estimate converges to x ≈ 3.14159265358979, the solution to the equation tan(x) = 0 to that many decimal places as well.

How to find the solution of equations to as many decimal places as possible?

To use Newton's method to solve the equation tan(x) = 0 with an initial estimate of xo = 3, we need to follow these steps:

1. Find the derivative of the function f(x) = tan(x): f'(x) = sec^2(x).
2. Use the formula for Newton's method: xn+1 = xn - f(xn)/f'(xn)
3. Substitute f(x) = tan(x) and f'(x) = sec^2(x) into the formula: xn+1 = xn - tan(xn)/sec^2(xn)
4. Plug in xo = 3 and use your calculator to find xn+1:
x1 = xo - tan(xo)/sec^2(xo) = 3 - tan(3)/sec^2(3) ≈ 3.1425465430743
x2 = x1 - tan(x1)/sec^2(x1) ≈ 3.14159265358979
x3 = x2 - tan(x2)/sec^2(x2) ≈ 3.14159265358979

We can see that the estimate converges to x ≈ 3.14159265358979, which is the value of pi to 14 decimal places. Therefore, we can estimate the solution to the equation tan(x) = 0 to that many decimal places as well.

Learn more about Newton's methods

brainly.com/question/30894507

#SPJ11

Which expressions are equivalent to b2c52b−2c12? Select all that apply

Answers

The "equivalent-expression" for the given expression "b²c⁵b¹ - 2c¹b²" is b²c(bc⁴ - 2).

An "Equivalent-Expression" is an expression which has the same-value as the original expression, but may look different. The two expressions are equivalent if they simplify to the same result.

We have to solve the expression : "b²c⁵b¹ - 2c¹b²",

To simplify this expression, we first combine the "like-terms" by adding the exponents of b and c;

= b²c⁵b¹ - 2c¹b²,

Now we add the exponents having the same-base;

= b²⁺¹c⁵ - 2b²c¹;

= b³c⁵ - 2b²c

= b²c(bc⁴ - 2).

Therefore, the required "equivalent-expression" is b²c(bc⁴ - 2).

Learn more about Expression here

https://brainly.com/question/30436325

#SPJ1

The given question is incomplete, the complete question is

Write an equivalent expression for the given expression "b²c⁵b¹ - 2c¹b²".

Zahra and some friends are going to the movies. At the theater, they sell a bag of popcorn for $3.50 and a drink for $5. How much would it cost if they bought 8 bags of popcorn and 5 drinks? How much would it cost if they bought
p bags of popcorn and d drinkss

Answers

Using basic mathematical procedures, we can determine that the total cost of the 8 bags of popcorn and 5 drinks is $53.

What do math operations entail?

An operation, in mathematics, is a mathematical function that transforms zero or more input values into a precisely defined output value.

The quantity of operands affects the operation's arity.

The rules that specify the order in which we should carry out the operations required to solve an equation are referred to as the order of operations.

PEMDAS stands for Parentheses, Exponents, Multiplication, Division, Addition, and Subtraction. (from left to right).

The whole price is thus:

Popcorn costs $3.50 for one bag.

One beverage costs $5.

Total cost for 5 beverages and 8 bags of popcorn:

(8 × 3.50) + (5 × 5) 28 + 25 $53

Therefore ,Using basic mathematical procedures, we can determine that the total cost of the 8 bags of popcorn and 5 drinks is $53.

To know more about mathematical operations visit:

brainly.com/question/28362686

#SPJ1

The appropriate response is provided below:

Zahra is going to the cinema with a few of her friends. A bag of popcorn costs $3.50 and a drink costs $5 at the theatre. How much would it cost if they purchased 5 beverages and 8 bags of popcorn?

Randy divides (2x4 – 3x3 – 3x2 7x – 3) by (x2 – 2x 1) as shown below. what error does randy make? x squared minus 2 x 1 startlongdivisionsymbol 2 x superscript 4 baseline minus 3 x cubed minus 3 x squared 7 x minus 3 endlongdivisionsymbol. minus 2 x superscript 4 baseline minus 4 x cubed 2 x squared to get a remainder of x cubed minus 5 x squared 7 x. minus x cubed minus 2 x squared x to get a remainder of negative 3 x squared 6 x minus 3. minus negative 3 x squared 6 x minus 3 to get a remainder of 0 and a quotient of 2 x squared x 3. he makes a subtraction error. he makes an error writing the constant term in the quotient. he makes an error choosing the x-term in the quotient. he makes an error rewriting the problem in long division.

Answers

By subtracting this from the dividend, the next step would be:

[tex](2x^4 - 3x^3 - 3x^2 + 7x - 3) - (-5x^3 + 10x^2 - 5x) = 2x^4 + 2x^3 - 13x^2 + 12x - 3[/tex]

This error occurs because he forgets to distribute the -2 in [tex]-2(x^2 - 2x + 1)[/tex]when subtracting from [tex]2x^4[/tex]. This leads to a mistake in the next step when he subtracts [tex]x^3 - 2x^2[/tex] from [tex]x^3 - 5x^2[/tex] to get [tex]-3x^2[/tex]instead of [tex]-3x^2 + 6x[/tex]. This error then leads to the incorrect constant term in the quotient.

Therefore, the error Randy makes is a subtraction error in the first step of the long division. It is important to pay attention to signs and distribute coefficients correctly when performing long division with polynomials.

To know more about remainder refer here:

https://brainly.com/question/30302805#

#SPJ11

Answer: A. x + 2

Step-by-step explanation:

Edge 2023

a cylinder has a radius of 3.8 meters it’s volume is 154 cubic meters

Answers

Answer:

h ≈ 3.39

Step-by-step explanation:

V = πr^2h

h = V/πr^2

h = 154/ π · 3.8^2

h ≈ 3.39472

Janelle has to solve this system of equations: 3x+5y=7 3x+5y=-4
She says, "I can tell just by looking that this system will have no solutions." What does she mean? How can she tell?

Answers

The system has no solution because the equations are parallel

What does she mean and How can she tell?

Janelle is correct in saying that the system of equations has no solution. She can tell by looking at the coefficients of the variables in the two equations.

Both equations have the same coefficients for x and y, which means that they are parallel lines in the xy-plane.

Since parallel lines never intersect, there are no values of x and y that would satisfy both equations simultaneously, meaning that the system has no solution.

Read more about system of equations at

https://brainly.com/question/13729904

#SPJ1

How do I solve this?

Answers

Step-by-step explanation:

you can solve cos(u) by

cos(u) = adjecent / hypotenes...general formula of cos

cos(u) = √44 / 12

cos(u) = 2√11 / 12 ..... √44 = √4×11 = 2√11

cos(u) = √11 / 6

u = cos^-1 ( √11 / 6 ) ..... divided both aide by cos ( multiple by cos invers )

u = 56.442 .... so we get it's angle

Answer:

[tex]cos(U)=\frac{\sqrt{11} }{6}[/tex]

Step-by-step explanation:

In a right triangle, the cosine of an angle is defined as the ratio of the adjacent side to the hypotenuse. Therefore, we have:

cos(U) = adjacent/hypotenuse = TU/SU

We are given that TU = sqrt(44) and SU = 12, so:

cos(U) = sqrt(44)/12

To simplify this expression, we can first factor 44 into 4 * 11, since 4 is a perfect square and a factor of 44:

cos(U) = sqrt(4 * 11) / 12

cos(U) = (sqrt (4) * sqrt (11)) / 12

cos (U) = (2 * sqrt (11)) / 12

Simplifying the fraction by dividing both the numerator and denominator by 2, we get:

cos(U) = sqrt(11)/6

Therefore, the exact value of cos(U) in simplest radical form is sqrt(11)/6

Furthermore, if you want another way to write the answer, dividing by 6 is the same as multiplying by 1/6 so you can do cos (U) = 1/6 * sqrt (11)

Although the other individual was correct that you use inverse trig (cos ^ -1) to find the measure of U, getting an exact answer requires us to leave it in simplest radical form since the number is so large and at best will yield an approximation if you don't keep it in simplest radical form.

Can you explain what is the horizontal tangent plane and how
should I use the tangent plane equation to answer this question,
thanks.
equation: f(a,b) + f(1)(x-a) + f(2)(y-b) = z

Answers

The value of the function at that point is equal to the z-coordinate of the point on the plane.

How to use the tangent plane equation to find the equation of a tangent plane?

A horizontal tangent plane is a plane that is parallel to the x-y plane and tangent to a surface at a point where the slope in the horizontal direction is zero.

To use the tangent plane equation to find a horizontal tangent plane, we need to find the partial derivatives of the function with respect to x and y, evaluate them at the point of interest, and check if they are both zero.

If they are both zero, then the tangent plane is horizontal and the equation simplifies to f(a,b) = z.

The tangent plane equation is given by:

f(a,b) + f(1)(x-a) + f(2)(y-b) = z

where (a,b) is the point where the tangent plane intersects the surface, and f(1) and f(2) are the partial derivatives of the function with respect to x and y, evaluated at (a,b).

To use this equation to find the horizontal tangent plane, we first find the partial derivatives f(1) and f(2), and evaluate them at the point where we want to find the tangent plane. If f(1) and f(2) are both zero at that point, then the tangent plane is horizontal and the equation simplifies to:

f(a,b) = z

This means that the value of the function at that point is equal to the z-coordinate of the point on the plane.

Learn more about tangent plane.

brainly.com/question/30885019

#SPJ11

Ben's Barbershop has a rectangular logo for their measuresb 7 1/5 feet long with an area that is exactly the maximum area allowed by thr building owner.



Create an equation that could be used to determine M, the unknown side length of the logo

Answers

An equation that could be used to determine M, the unknown side length of the logo is X = (36/5) x M

Let's assume that the unknown side length of the logo is 'M'. The logo is a rectangle, and the area of a rectangle is given by multiplying its length and width. Since we know the length of the logo is 7(1)/(5) feet, we can write the equation:

A = L x W

where A is the area of the logo, L is the length of the logo, and W is the width of the logo.

Substituting the given values, we get:

A = (7(1)/(5)) x M

or

A = (36/5) x M

Now, we know that the area of the logo is exactly the maximum area allowed by the building owner. Let's assume this maximum area is 'X'. So, we can write another equation:

A = X

Combining both equations, we get:

X = (36/5) x M

This is the required equation that could be used to determine the unknown side length 'M' of the logo if we know the maximum area allowed by the building owner 'X'.

To know more about equation, refer to the link below:

https://brainly.com/question/30137119#

#SPJ1

A squirrel and a chipmunk are each collecting pinon nuts for the winter. They have each saved an equal amount. How many pinon nuts would the squirrel have to give the chipmunk so that the chipmunk would have ten more pinon nuts than the squirrel?



Please help me

Answers

Let x be the number of pinon nuts each animal has collected. To make the chipmunk have ten more pinon nuts than the squirrel, the squirrel would have to give the chipmunk 10 pinon nuts.

So, after the exchange, the squirrel would have x - 10 pinon nuts, and the chipmunk would have x + 10 pinon nuts.

Since they are each giving an equal amount, the total number of pinon nuts remains the same. Therefore, we can set up the equation:

x + (x - 10) = 2x - 10

Simplifying and solving for x, we get:

2x - 10 = 2x

-10 = 0

This is a contradiction, so there is no solution that satisfies the conditions of the problem.

Therefore, the problem is not well-defined and there is no answer.

To know more about chipmunk refer here

https://brainly.com/question/30368709#

#SPJ11

Consider the following piecewise-defined function. F(x) = {22

- 5,x < 3

(2x + 5,x > 3

Find f(-4)

Answers

For the piecewise-defined function, f(-4) = 42.

The given function is a piecewise-defined function, which means that it is defined differently depending on the value of x. In this case, we have two different formulas for the function depending on whether x is less than or greater than 3. For values of x less than 3, the function is given by f(x) = 22 - 5x, while for values of x greater than 3, the function is given by f(x) = 2x + 5.

To find f(-4), we need to determine which part of the function applies to the value of x = -4. Since -4 is less than 3, we use the first part of the function, which gives us f(-4) = 22 - 5(-4) = 22 + 20 = 42. This means that if x is equal to -4, the function f(x) evaluates to 42.

Piecewise-defined functions can be useful in modeling real-world problems where the relationship between variables changes depending on certain conditions or constraints. By defining the function differently depending on the value of x, we can more accurately capture the behavior of the system being modeled.

In this case, the function could be used to model a situation where the value of a variable has different relationships to other variables depending on whether it is less than or greater than a certain threshold value.

To know more about piecewise-defined function, refer to the link below:

https://brainly.com/question/12561612#

#SPJ11

Twenty people each choose a number from a choice of, 1,2,3,4 or 5. the mode is larger than the median. the median is larger than the mean


fill in a set of possible frequency

Answers

To satisfy the conditions that the mode is larger than the median, and the median is larger than the mean, one possible set of frequencies is 1 person chooses 1, 3 people choose 2, 4 people choose 3, 1 person chooses 4 and 11 people choose 5 This results in a mode of 5, a median of 4, and a mean of approximately 3.75.

Since we are given that the mode is larger than the median, that means that at least 11 people must choose the same number. Let's assume that 11 people choose the number 5.

Now, since the median is larger than the mean, we want to make sure that the remaining 9 people choose numbers that are smaller than 5. If they all choose 1, 2, or 3, then the median will be 3, which is larger than the mean. Therefore, we need to make sure that at least one person chooses 4.

So one possible set of frequencies could be

1 person chooses 1

3 people choose 2

4 people choose 3

1 person chooses 4

11 people choose 5

This set of frequencies gives us a mode of 5 (since 11 people choose 5), a median of 4 (since the middle value is 4), and a mean of

(11 + 32 + 43 + 14 + 11*5) / 20 = 3.7

Since the median is larger than the mean, this set of frequencies satisfies all the given conditions.

To know more about mode:

https://brainly.com/question/30891252

#SPJ4

What is the value of x in the solution to this system of equations 5x-4y=27

y=2x+3

Answers

The value of x in the solution to this system of equations 5x - 4y = 27 and y = 2x + 3 is -13.

To find the value of x in this system of equations, we can use substitution method to find the its solution. Start by isolating x in one of the equations and then substituting that value into the other equation.

Let's start by isolating x in the second equation:

y = 2x + 3

Subtracting 3 from both sides:

y - 3 = 2x

Dividing both sides by 2:

(1/2)y - (3/2) = x

Now we can substitute this expression for x into the first equation:

5x - 4y = 27

5((1/2)y - (3/2)) - 4y = 27

Simplifying:

(5/2)y - 15/2 - 4y = 27

Combining like terms:

-(3/2)y = 69/2

Dividing by -(3/2):

y = -23

Now we can substitute this value of y back into the expression we found for x:

x = (1/2)y - (3/2)

x = (1/2)(-23) - (3/2)

x = -13

Therefore, the solution to this system of equations is x = -13, y = -23.

Learn more about substitution method here: https://brainly.com/question/30339603

#SPJ11

I need help with this one

solve for x

Answers

Answer:

x = 2

Step-by-step explanation:

A secant is a straight line that intersects a circle at two points.

A segment is part of a line that connects two points.

According to the Intersecting Secants Theorem, the product of the measures of one secant segment and its external part is equal to the product of the measures of the other secant segment and its external part.

The given diagram shows two secant segments that intersect at an exterior point.  

One secant segment is (6x - 1 + 7) and its external part is 7.The other secant segment is (x + 3 + 9) and its external part is 9.

Therefore, according to the Intersecting Secants Theorem:

[tex](6x-1+7) \cdot 7=(x+3+9) \cdot 9[/tex]

Solve for x:

[tex]\begin{aligned}(6x+6) \cdot 7&=(x+12) \cdot 9 \\42x+42&=9x+108\\42x+42-9x&=9x+108-9x\\33x+42&=108\\33x+42-42&=108-42\\33x&=66\\33x\div33&=66\div33\\x&=2 \end{aligned}[/tex]

Therefore, the value of x is x = 2.

Learn more about intersecting secants here:

https://brainly.com/question/31338770

A home buyer is financing a house for $135,950. The buyer has to pay $450 plus 1.15% for a brokerage fee. How much are the mortgage brokerage fees?

$2,489.25
$2,013.43
$2,018.60
$2,031.43

Answers

Answer: $2,013.43

Step-by-step explanation:

$135,950 x 1.15% = 1,563.425

Round to $1,563.43

Add in $450

$1,563.43 + $450 = $2,013.43

The answer is 2,013.43

Dans une boite il ya 12 boules vertes et 6 boules bleues quelle est la proportion de boules vertes dans cette boite

Answers

La proportion de boules vertes dans cette boîte est de 2/3.

How to calculate the proportion of green balls in the box?

Pour déterminer la proportion de boules vertes dans cette boîte, nous devons comparer le nombre de boules vertes au nombre total de boules dans la boîte.

Le nombre total de boules dans la boîte est la somme des boules vertes et des boules bleues, soit 12 + 6 = 18 boules.

Maintenant, pour calculer la proportion de boules vertes, nous divisons le nombre de boules vertes par le nombre total de boules.

Proportion de boules vertes = Nombre de boules vertes / Nombre total de boules

Proportion de boules vertes = 12 / 18

Simplifiant cette fraction, nous obtenons :

Proportion de boules vertes = 2/3

La proportion de boules vertes dans cette boîte est donc de 2/3 ou environ 66.67%.

Learn more about proportion

brainly.com/question/21126582

#SPJ11

Let F(X, y, 2) = 3z^2xi + (y^3 + tan(2)J + (3x^2z + 1y^2)k. Use the Divergence Theorem to evaluate /s. F. dS where S is the top half of the sphere x^2 + y^2 + z^2 = 1 oriented upwards. s/sF. ds =SIF. ds =

Answers

The given problem involves evaluating the surface integral of the vector field F(X, y, 2) over the top half of a sphere x^2 + y^2 + z^2 = 1, oriented upwards, using the Divergence Theorem.

The Divergence Theorem states that the flux of a vector field F through a closed surface S is equal to the triple integral of the divergence of F over the region enclosed by S.

In this problem, the given vector field F(X, y, z) is F(X, y, 2) = 3z^2xi + (y^3 + tan(2)J + (3x^2z + 1y^2)k.

The surface S is the top half of the sphere x^2 + y^2 + z^2 = 1, oriented upwards. This means that z is positive on S, and the normal vector points in the positive z-direction.

To use the Divergence Theorem, we need to find the divergence of F. The divergence of F is given by div(F) = ∂Fx/∂x + ∂Fy/∂y + ∂Fz/∂z, where ∂Fx/∂x, ∂Fy/∂y, and ∂Fz/∂z are the partial derivatives of F with respect to x, y, and z, respectively.

Taking the partial derivatives of F with respect to x, y, and z, we get:

∂Fx/∂x = 6xz

∂Fy/∂y = 3y^2 + 2y

∂Fz/∂z = 0

So, the divergence of F is: div(F) = 6xz + 3y^2 + 2y

Now, we can apply the Divergence Theorem, which states that the surface integral of F over S is equal to the triple integral of the divergence of F over the region enclosed by S.

The triple integral of the divergence of F over the region enclosed by S can be written as: ∫∫∫ div(F) dV, where dV is the volume element.

Since the given problem asks for the surface integral of F over S, we only need to consider the part of the triple integral that involves the surface S.

The surface integral of F over S can be written as: ∫∫ F · dS, where dS is the outward-pointing normal vector on S and · represents the dot product.

The dot product F · dS can be expressed as: Fx * dSx + Fy * dSy + Fz * dSz, where Fx, Fy, and Fz are the components of F, and dSx, dSy, and dSz are the components of the outward-pointing normal vector on S.

Since the normal vector on S points in the positive z-direction, we have dSx = 0, dSy = 0, and dSz = 1.

Substituting the components of F and the components of dS into the expression for the dot product, we get: Fx * dSx + Fy * dSy + Fz * dSz = (3z^2x)(0) + (y^3 + tan(2)J + (3x^2z +

For more questions like Integral click the link below:

https://brainly.com/question/22008756

#SPJ11

Mr. Larson, a math teacher, assigned his students a project to do in pairs. He recorded the


grade each pair earned.


Math project grades


92 77 97 70 96 75


73


84


71


87


80


86


100


95


Which box plot represents the data?


Math project grades


50


60


70


80


90


100


Math project grades


50


60


70


80


90


100

Answers

The box plot that would represent the data recorded by Mr. Larson would be B. Second box plot.

How to find the box plot ?

To find the correct box plot of the data recorded by Mr. Larson, the math teacher, first order the grades from lowest to highest :

70, 71, 73, 75, 77, 80, 84, 86, 87, 92, 95, 96, 97, 100

There are 14 grades which means that the median position would be the 7th and 8th grades average :

= ( 84 + 86 ) / 2

= 170 / 2

= 85

The position of Q3 would be:

= ( n + 1 ) x 75 %

= ( 14 + 1 ) x 75 %

= 11 th position which is 95

The correct box plot is therefore the second box plot which shows the Q3 as 95.

Find out more on box plots at https://brainly.com/question/27849170

#SPJ1

I need help solving ration expressions

Answers

The simplified form of the given expression is (x-7)/3x.

The given expression is (2x²-8x-42)/6x² ÷ (x²-9)/(x²-3x)

Here, (x²-4x-21)/3x² ÷ (x-3)(x+3)/x(x-3)

= (x²-4x-21)/3x² ÷ (x+3)/x

= (x²-4x-21)/3x² × x/(x+3)

= (x²-4x-21)/3x × 1/(x+3)

= (x²-4x-21)/3x(x+3)

= (x²-7x+3x-21)/3x(x+3)

= [x(x-7)+3(x-7)]/3x(x+3)

= (x-7)(x+3)/3x(x+3)

= (x-7)/3x

Therefore, the simplified form of the given expression is (x-7)/3x.

To learn more about an expression visit;

https://brainly.com/question/28170201.

#SPJ1

Other Questions
This box plot shows scores on a recent math test in a sixth grade class. Identify at least three things that you can infer from the box plot about the distributions center, variability, and spread. Why are the languages, Czech, Slovene and Slovak considered slightly endangered by some linguists? The tables represent hat sizes measured in inches for two softball teams.Pelicans21.5 25 2221 22 2322.5 24 21.522 23.5 2223.5 22 24.5Falcons22.5 20 23.521 24 2220.5 21.5 2323 22.5 2124 22 24.5Which team has the largest overall size hat for their playersThe tables represent hat sizes measured in inches for two softball teams.Pelicans21.5 25 2221 22 2322.5 24 21.522 23.5 2223.5 22 24.5Falcons22.5 20 23.521 24 2220.5 21.5 2323 22.5 2124 22 24.5Which team has the largest overall size hat for their players? Determine the best measure of center to compare and explain your answer. Falcons; they have a larger median value of 22.5 inches Pelicans; they have a larger median value of 22 inches Falcons; they have a larger mean value of about 22 inches Pelicans; they have a larger mean value of about 23 inches? Determine the best measure of center to compare and explain your answer. Falcons; they have a larger median value of 22.5 inches Pelicans; they have a larger median value of 22 inches Falcons; they have a larger mean value of about 22 inches Pelicans; they have a larger mean value of about 23 inches In circle N with \text{m} \angle MQP= 44^{\circ}mMQP=44 , find the angle measure of minor arc \stackrel{\Large \frown}{MP}. MP . M P N Q State how many terms are in each algebraic expression: (a) -112y2 ____________________ [1mark](b) 7x2 + 5y 9xy + 3 __________________ [1mark] Please upload a picture of a piece of paper with the problem worked out, and draw the graph for extra points, there will be 6 of these, so go to my profile and find the rest, and do the same, for extra points. solve the system using the ELIMINATION method. The NaOH solution was made from 142. 1 g NaOH, dissolved in water and diluted to 1000. 0 +/- 0. 6 mLWhat is the molarity of the NaOH solution prepared to react with the pennies?What was the pH of the solution? a 450.0-g block is attached to a spring of spring constant 45 n/m, and is in simple harmonic motion on a frictionless surface with an amplitude of 7.5 cm. a second identical block, moving at 12 m/s in line with the spring, strikes the first one when the spring is at its maximum extension. the two blocks stick together. what is the amplitude of the resulting oscillation? i need help its due in 2 hours Write the following Babylonian numeral as a Hindu-Arabic numeral. What is the area of this figure?8 m6 m5 m2 m13 m8 m We collected data from 9th and 10th. 9th grade students were 45% of the responses and 10th grade were therest. Of the 9th graders 31% said they did like the school lunches and 42% of the 10th graders said they did likethe school lunches. Find the probability that if we chose a student at random that they would not like the schoollunches. what is the volume of this triangular prism? Then tell us what elements of the Promotional Mix did the company use or uses for that product or service? Also, is product or service communication integrated? How or how not? CANADAIn early 2022, the GDP per capita of the United States was $59,312. How does this compare with your selected country? What does it tell you about standard of living in your selected country versus the United States? Respond to these questions using at least three to five sentences. Part AAlex has \$ 30,000$30,000 in his savings account that earns 10\%10% annually. How much interest will he earn in one year? Interest == \$$ Part BIf Alex spends 20\%20% of the interest received on buying furniture for his new house, what amount did he spent on furniture? Proteins differ from carbohydrates in that, besides the elements present in sugar and starches, they also contain nitrogen and in most cases sulphur. phosphorus and iron are also found in some proteins. the importance of nitrogen content commonly varies from 10 to 18 per cent. proteins are found in foods of both animal and plant origin. meat, eggs, milk, nuts and certain of cereal foods are high in protein. all those foods contain more than one protein. milk, for example, contains the proteins, lactalbumin and casein, as well as others of lesser importance.proteins are complex in nature, they are composed of a number of amino acids or "building blocks", linked together, and also of amino acids linked with other substances. proteins vary in the kind and number of amino acids present. certain amino acids the body is unable to build. these are known as essential amino acids and must be supplied by the foods eaten.variation in the quality of proteins has resulted in their classification as complete, partially complete, and incomplete, based upon their ability to support normal growth and to maintain life even if it is the only protein in the diet. a partially complete protein is one that maintain life but does not support normal growth. an incomplete protein, although valuable in the diet, will by itself neither support normal growth nor maintain life. most animal proteins are complete. plant proteins are more or less incomplete, although the soya bean and the wheat grain contain proteins that are complete. valuable sources of protein are: cereals and cereal products, cheese, eggs, fish, meat, milk, nuts. 1.what is the composition of proteins?2.what foods are valuable sources of proteins?3.what is the basis of protein classification? according to porter, a firm in a competitive marketplace with no clear generic competitive strategy (or change back and forth between the two generic competitive strategies), which leads to no clear competitive advantage in comparison to competitors, is called please help me, ill give brainlist and shoutouts on socials. Worldwide, more than 4 billion people carry cell phones. Because people tend to replace their phones every 9 to 18 months, many broken and unwanted devices end up in landfills, where they can release toxins into the environment. What is the term used to describe these kinds of throwaway toxic electronics?.